Search found 107 matches


You are right Vemuri. I was not careful. It should be 119. I'll edit on my posting and not to mislead people.

Thanks!

by pakaskwa

Sun Apr 26, 2009 11:13 pm
Forum: Problem Solving
Topic: How Many 2 Elemen Subsets of.....
Replies: 6
Views: 1692

1st, we need to determine if it's a "combination" or "permutation" question. In the question stem, it says "pair of elements 7 and 9". A pair means "7,9" and "9,7" are the same. So it's a combination issue. The order of 7 and 9 doesn't matter. 2nd, i...

by pakaskwa

Sat Apr 25, 2009 2:43 pm
Forum: Problem Solving
Topic: How Many 2 Elemen Subsets of.....
Replies: 6
Views: 1692

IMO, A. Stmt 1 AD=AC --> angle ADC=ACD --> angle ADE=ACB and we know that angle EAD=BAC --> AED=ABC So AB=AE. Stmt 1 SUFF. Stmt 2 Let angle EAD=DAC=CAB=x Since AC=CB --> angle ABC=CAB=x --> angle ACB=180-ABC-CAB=180-2x --> angle ACD=180-ACB=180-(180-2x)=2x --> angle ADC=180-3x --> angle ADE=3x --> a...

by pakaskwa

Sat Apr 25, 2009 2:28 pm
Forum: Data Sufficiency
Topic: angles and sides...
Replies: 2
Views: 1150

IMO it's B.

To pick out any 2 items out of 3 items, there are 2C3 possible ways.

To pick out 2 selected items out of 3 items, there are 2C2 possible ways.

Therefore, to pick out any 2 items out of 3 items, but NOT including those 2 selected items are:
2C3-2C2=3-1=2

by pakaskwa

Sat Apr 25, 2009 2:06 pm
Forum: Problem Solving
Topic: How Many 2 Elemen Subsets of.....
Replies: 6
Views: 1692

a) There's only 1 first prize in 30 tickets. If the boy has 1 ticket, his probability is 1/30. When he has 3 tickets, it's 3*1/30=1/10. b) To win first prize, his probability is 1/10; to win 2nd prize, probability is 2/29 (29 tickets left). To win both, probability is 1/10*2/29=1/145 c) The probabil...

by pakaskwa

Sat Apr 25, 2009 1:51 pm
Forum: Problem Solving
Topic: raffle: probability
Replies: 4
Views: 5647

IMO, E. It's a bit tricky to understand "divided successively". IMO, it means that, number x is divided by 4 first, quotient is n, remainder is 1. Then n is divided by 5, quotient is m, remainder is 4. (m and n are positive ingeters.) If my understanding is correct, number x = (5m+4)4n+1 =...

by pakaskwa

Tue Apr 21, 2009 12:02 pm
Forum: Problem Solving
Topic: successive division
Replies: 3
Views: 14412

IMO choice E. Let x be the number, we can get x=31n+29. Since 31 is an odd number, 31n could be odd or even (it's odd when n is odd, it's even when n is even). So 31n+29 can be either odd or even. We can easily deduct: when odd numbers are divided by even number 16, there must be remainders; when ev...

by pakaskwa

Tue Apr 21, 2009 11:38 am
Forum: Problem Solving
Topic: confused divisor
Replies: 2
Views: 1295

Stmt 2:
If you draw a line, and arrange point A, B, and C in following sequence:

1. A-C-B, distance AC is 5, distance BC is 20. (distance AB is 25) C is between A and B.
2. C-A-B, AC is 5, BC is 20. (but AB is 15). C is NOT between A and B.

So stmt 2 is not sufficient.

by pakaskwa

Tue Apr 21, 2009 11:27 am
Forum: Problem Solving
Topic: Point A lies to the left of point B
Replies: 1
Views: 987

Personally I dont' think any GMAT materials are close enough to real test, including Princeton, MCGMAT, Manhatten, Kaplan, etc. And trust me I tried quite a few of them. The best materials out there is OG. Get your hands on all OGs, like OG10-12. Math: most of the math books are relatively easier th...

by pakaskwa

Tue Apr 21, 2009 11:15 am
Forum: GMAT Strategy
Topic: What to do? Gmat Soon
Replies: 2
Views: 1350

From question stem we can get:
g(2)=(2^2+2)/(2-1)=6 --> 1/g(2)=1/6
g(x)=(x^2+2)/(x-1) --> 1/g(x)=(x-1) /(x^2+2)

[1/g(2)]*[1/g(x)] = (x-1) /6(x^2+2)

Choose D.

by pakaskwa

Tue Apr 21, 2009 10:49 am
Forum: Problem Solving
Topic: GMATPrep function
Replies: 1
Views: 1094

You are right CR. I made a mistake with Stmt 2. From stmt 2, we can get x(x+1)(x-1)<0 Then there are 3 possiblities: 1. x>0, x+1>0, x-1<0 --> 0<x<1 2. x>0, x+1<0, x-1>0 --> x>1 (out) 3. x<0, x+1>0, x-1>0 --> no solution So from stmt 2 we know that x>0. SUFF! Choice is D! I'll edit on my previous pos...

by pakaskwa

Thu Apr 16, 2009 3:02 pm
Forum: Data Sufficiency
Topic: Is X>0?
Replies: 20
Views: 3669

Hi CR and all, Thanks for your help. It's trickier than it looks eh? CR, my approach is similar to yours, yet I got different answer. From stmt 1 , we can get x(x-1)<0 so there are two solutions: 1. x<0, x>1, or 2. x>0, x<1 --> 0<x<1 The question stem told us that -1<x<1, so 1st solution is out. 2nd...

by pakaskwa

Thu Apr 16, 2009 1:24 pm
Forum: Data Sufficiency
Topic: Is X>0?
Replies: 20
Views: 3669

The OA is C. I just did the question yesterday on GMATPrep. I chose D, and I was wrong.

by pakaskwa

Wed Apr 15, 2009 2:29 pm
Forum: Data Sufficiency
Topic: GMAT Prep Prac 1 - Q19
Replies: 15
Views: 7764

Is X>0?

If -1<x<1, is x>0?
(1) x^2<x
(2) x^3<x

A friend of mine asked me this question, and we don't know the OA.

by pakaskwa

Wed Apr 15, 2009 2:20 pm
Forum: Data Sufficiency
Topic: Is X>0?
Replies: 20
Views: 3669

I went for E too. I thought "its" is ambiguous pronoun here.

I understand the tense needed here is present perfect. But I can't tell which error is more serious in the eyes of GMAC... Or maybe "its" here is not an error since it can only refer to P.R. China?

by pakaskwa

Sun Apr 12, 2009 2:06 pm
Forum: Sentence Correction
Topic: 1000sc# 283
Replies: 9
Views: 9327